For those who are good in geometry

Rette, triangoli, cerchi, poliedri, ...
Rispondi
timothy6
Messaggi: 18
Iscritto il: 12 set 2007, 21:03

For those who are good in geometry

Messaggio da timothy6 »

We have an angle (less than 180 degree) with it's top in point P. We also have point A inside the angle (interior). Points X and Y are laying on different legs of this angle and |PX| = |PY|.

We must prove that when the sum: |AX| + |AY| is the smallest (minimum), the equation belove is correct:

Angle XAP is equal to angle YAP.



P.S Point A is "standing still", and can be anywhere inside the triangle. We only move X and Y points

Someone has an Idea how to solve it?
I am trying to solve it for month and still can't do it!
Sepp
Messaggi: 87
Iscritto il: 01 gen 1970, 01:00
Località: Vicenza

Messaggio da Sepp »

Questo ragazzo polacco tenta di fare il furbo! :) I due problemi che ha postato (uno qui, l'altro in algebra) sono del I round delle olimpiadi polacche, in corso fino al 8 Ottobre e per le quali è vietato "farsi aiutare". Sarebbe quindi scorretto rispondere con una soluzione.
pic88
Messaggi: 741
Iscritto il: 16 apr 2006, 11:34
Località: La terra, il cui produr di rose, le dié piacevol nome in greche voci...

Messaggio da pic88 »

Sepp ha scritto:Questo ragazzo polacco tenta di fare il furbo! :) I due problemi che ha postato (uno qui, l'altro in algebra) sono del I round delle olimpiadi polacche, in corso fino al 8 Ottobre e per le quali è vietato "farsi aiutare". Sarebbe quindi scorretto rispondere con una soluzione.
This Polish guy is a cheat! :) The problems he posted (one is here, the other one in the section "algebra") belong to the first round of Polish olympiad, which last until October, and for which "asking help" is forbidden. So it would be unfair to answer with a solution. :P
Avatar utente
FeddyStra
Messaggi: 403
Iscritto il: 19 set 2006, 15:34
Località: 45° 7' 19.2'' N 7° 23' 20.1'' E

Messaggio da FeddyStra »

pic88 ha scritto:... which last until October...
... which lasts until October...
:P
[quote="julio14"]Ci sono casi in cui "si deduce" si può sostituire con "è un'induzione che saprebbe fare anche un macaco", ma per come hai impostato i conti non mi sembra la tua situazione...[/quote][quote="Tibor Gallai"]Ah, un ultimo consiglio che risolve qualsiasi dubbio: ragiona. Le cose non funzionano perché lo dico io o Cauchy o Dio, ma perché hanno senso.[/quote]To understand recursion, you fist need to understand recursion.
[tex]i \in \| al \| \, \pi \, \zeta(1)[/tex]
pic88
Messaggi: 741
Iscritto il: 16 apr 2006, 11:34
Località: La terra, il cui produr di rose, le dié piacevol nome in greche voci...

Messaggio da pic88 »

Scommetto che se avessi fatto un errore di ortografia in italiano non me lo avresti segnalato :D
Comunque al massimo dovevo dire "Olympiads" visto che il testo originale era al plurale..
Avatar utente
edriv
Messaggi: 1638
Iscritto il: 16 feb 2006, 19:47
Località: Gradisca d'Isonzo
Contatta:

Messaggio da edriv »

Immagine

Set PX = PY = d. Then, by Ptolemy's inequality applied on the quarilateral AXPY:
$ ~ AX \cdot PY + AY \cdot PX \ge AP \cdot XY $
$ ~ AX + AY \ge AP \cdot \frac{XY}d $
Note that, since $ ~ \frac{XY}d $ is constant, so is the right hand side of our inequality as X,Y move. We have equality when AXPY is a cyclic quadrilateral, i.e., when $ ~ \angle XAP = \angle PAY $.
Rispondi